Difference between revisions of "2008 AMC 8 Problems/Problem 9"

(Created page with "==Problem 9== In <math>2005</math> Tycoon Tammy invested <math>100</math> dollars for two years. During the the first year her investment suffered a <math>15\%</math> loss, but d...")
 
m (Problem)
(3 intermediate revisions by 3 users not shown)
Line 1: Line 1:
==Problem 9==
+
==Problem==
In <math>2005</math> Tycoon Tammy invested <math>100</math> dollars for two years. During the the first year
+
In <math>2005</math> Tycoon Tammy invested <math>100</math> dollars for two years. During the first year
 
her investment suffered a <math>15\%</math> loss, but during the second year the remaining
 
her investment suffered a <math>15\%</math> loss, but during the second year the remaining
 
investment showed a <math>20\%</math> gain. Over the two-year period, what was the change
 
investment showed a <math>20\%</math> gain. Over the two-year period, what was the change
Line 11: Line 11:
 
\textbf{(E)}\  5\%\text{ gain}</math>
 
\textbf{(E)}\  5\%\text{ gain}</math>
  
 +
==Solution==
 +
After the <math>15 \%</math> loss, Tammy has <math>100 \cdot 0.85 = 85</math> dollars. After the <math>20 \%</math> gain, she has <math>85 \cdot 1.2 = 102</math> dollars. This is an increase in <math>2</math> dollars from her original <math>100</math> dollars, a <math>\boxed{\textbf{(D)}\ 2 \%\ \text{gain}}</math>.
 
==See Also==
 
==See Also==
 
{{AMC8 box|year=2008|num-b=8|num-a=10}}
 
{{AMC8 box|year=2008|num-b=8|num-a=10}}
 +
{{MAA Notice}}

Revision as of 20:51, 7 February 2018

Problem

In $2005$ Tycoon Tammy invested $100$ dollars for two years. During the first year her investment suffered a $15\%$ loss, but during the second year the remaining investment showed a $20\%$ gain. Over the two-year period, what was the change in Tammy's investment?

$\textbf{(A)}\  5\%\text{ loss}\qquad \textbf{(B)}\ 2\%\text{ loss}\qquad \textbf{(C)}\ 1\%\text{ gain}\qquad \textbf{(D)}\ 2\% \text{ gain} \qquad \textbf{(E)}\  5\%\text{ gain}$

Solution

After the $15 \%$ loss, Tammy has $100 \cdot 0.85 = 85$ dollars. After the $20 \%$ gain, she has $85 \cdot 1.2 = 102$ dollars. This is an increase in $2$ dollars from her original $100$ dollars, a $\boxed{\textbf{(D)}\ 2 \%\ \text{gain}}$.

See Also

2008 AMC 8 (ProblemsAnswer KeyResources)
Preceded by
Problem 8
Followed by
Problem 10
1 2 3 4 5 6 7 8 9 10 11 12 13 14 15 16 17 18 19 20 21 22 23 24 25
All AJHSME/AMC 8 Problems and Solutions

The problems on this page are copyrighted by the Mathematical Association of America's American Mathematics Competitions. AMC logo.png